K
Khách

Hãy nhập câu hỏi của bạn vào đây, nếu là tài khoản VIP, bạn sẽ được ưu tiên trả lời.

18 tháng 8 2019

By Cauchy-Schwarz, we have:

\(VT\ge\frac{\left(a^3+b^3+c^3\right)^2}{2\left(a^3+b^3+c^3\right)+a^2b+b^2c+c^2a}\)

We will prove: \(a^2b+b^2c+c^2a\le a^3+b^3+c^3\)

\(\Leftrightarrow a^2b+b^2c+c^2a+3abc\le a^3+b^3+c^3+3abc\)

By Schur, we have: \(RHS\ge ab\left(a+b\right)+bc\left(b+c\right)+ca\left(a\right)\)

So we're only need to prove: \(ab\left(a+b\right)+bc\left(b+c\right)+ca\left(c+a\right)\ge a^2b+b^2c+c^2a+3abc\)

\(\Leftrightarrow ab^2+bc^2+ca^2\ge3abc\)

It is true by AM-GM ineq', so we have Q.E.D.

P/s: Em thử giải bài này bằng tiếng Anh (để tự luyện kĩ năng tiếng anh, tí em giải lại theo tiếng việt)

18 tháng 8 2019

Ấy nhầm:V

By Schur, we have \(RHS\ge ab\left(a+b\right)+bc\left(b+c\right)+ca\left(c+a\right)\)

So we're only need to prove \(ab\left(a+b\right)+bc\left(b+c\right)+ca\left(c+a\right)\ge a^2b+b^2c+c^2a\)

Còn lại y chang:v

3 tháng 7 2019

Ta có \(\frac{a^3}{b}+\frac{b^3}{c}+\frac{c^3}{a}\ge\frac{\left(a^2+b^2+c^2\right)^2}{ab+bc+ac}\ge\frac{\left(ab+bc+ac\right)^2}{ab+bc+ac}=ab+bc+ac\left(1\right)\)

Áp dụng bất đẳng thức buniacoxki ta có :

\(\left(\frac{a^5}{b^3}+\frac{b^5}{c^3}+\frac{c^5}{a^3}\right)\left(ab+bc+ac\right)\ge\left(\frac{a^3}{b}+\frac{b^3}{c}+\frac{c^3}{a}\right)^2\)

Kết hợp với (1)

=> \(\frac{a^5}{b^3}+\frac{b^5}{c^3}+\frac{c^5}{a^3}\ge\frac{a^3}{b}+\frac{b^3}{c}+\frac{c^3}{a}\)(ĐPCM)

Dấu bằng xảy ra khi a=b=c

2 tháng 7 2019

Nghe mùi holder ?

7 tháng 8 2019

Áp dụng BĐT cosi ta có

\(\frac{1}{a^3}+\frac{1}{a^3}+\frac{1}{b^3}\ge\frac{3}{a^2b}\)\(\frac{1}{b^3}+\frac{1}{b^3}+\frac{1}{c^3}\ge\frac{3}{b^2c}\)\(\frac{1}{c^3}+\frac{1}{c^3}+\frac{1}{d^3}\ge\frac{3}{c^2d}\)

\(\frac{1}{d^3}+\frac{1}{d^3}+\frac{1}{a^3}\ge\frac{3}{d^2a}\)

Cộng các BĐt trên ta có 

\(\frac{1}{a^3}+\frac{1}{b^3}+\frac{1}{c^3}+\frac{1}{d^3}\ge\frac{1}{a^2b}+\frac{1}{b^2c}+\frac{1}{c^2d}+\frac{1}{d^2a}\)(1)

Áp dụng BĐT buniacoxki ta có

\(\left(\frac{a^2}{b^5}+\frac{b^2}{c^5}+\frac{c^2}{d^5}+\frac{d^2}{a^5}\right)\left(\frac{1}{a^2b}+\frac{1}{b^2c}+\frac{1}{c^2d}+\frac{1}{d^2a}\right)\ge \left(\frac{1}{a^3}+\frac{1}{b^3}+\frac{1}{c^3}+\frac{1}{d^3}\right)^2\)

Kết hợp với (1)  ta được ĐPCM

Dấu bằng xảy ra khi a=b=c

2 tháng 7 2019

Áp dụng Svac + Cô-si 3 số được

\(\frac{a^5}{bc}+\frac{b^5}{ca}+\frac{c^5}{ab}=\frac{a^6}{abc}+\frac{b^6}{abc}+\frac{c^6}{abc}\ge\frac{\left(a^3+b^3+c^3\right)^2}{3abc}\ge\frac{\left(a^3+b^3+c^3\right)^2}{a^3+b^3+c^3}=VP\left(đpcm\right)\)

"=" tại a = b = c

9 tháng 8 2019

2) Theo nguyên lí Dirichlet, trong ba số \(a^2-1;b^2-1;c^2-1\) có ít nhất hai số nằm cùng phía với 1.

Giả sử đó là a2 - 1 và b2 - 1. Khi đó \(\left(a^2-1\right)\left(b^2-1\right)\ge0\Leftrightarrow a^2b^2-a^2-b^2+1\ge0\)

\(\Rightarrow a^2b^2+3a^2+3b^2+9\ge4a^2+4b^2+8\)

\(\Rightarrow\left(a^2+3\right)\left(b^2+3\right)\ge4\left(a^2+b^2+2\right)\)

\(\Rightarrow\left(a^2+3\right)\left(b^2+3\right)\left(c^2+3\right)\ge4\left(a^2+b^2+1+1\right)\left(1+1+c^2+1\right)\) (2)

Mà \(4\left[\left(a^2+b^2+1+1\right)\left(1+1+c^2+1\right)\right]\ge4\left(a+b+c+1\right)^2\) (3)(Áp dụng Bunhicopxki và cái ngoặc vuông)

Từ (2) và (3) ta có đpcm.

Sai thì chịu

9 tháng 8 2019

Xí quên bài 2 b:v

b) Không mất tính tổng quát, giả sử \(\left(a^2-\frac{1}{4}\right)\left(b^2-\frac{1}{4}\right)\ge0\)

Suy ra \(a^2b^2-\frac{1}{4}a^2-\frac{1}{4}b^2+\frac{1}{16}\ge0\)

\(\Rightarrow a^2b^2+a^2+b^2+1\ge\frac{5}{4}a^2+\frac{5}{4}b^2+\frac{15}{16}\)

Hay \(\left(a^2+1\right)\left(b^2+1\right)\ge\frac{5}{4}\left(a^2+b^2+\frac{3}{4}\right)\)

Suy ra \(\left(a^2+1\right)\left(b^2+1\right)\left(c^2+1\right)\ge\frac{5}{4}\left(a^2+b^2+\frac{1}{4}+\frac{1}{2}\right)\left(\frac{1}{4}+\frac{1}{4}+c^2+\frac{1}{2}\right)\)

\(\ge\frac{5}{4}\left(\frac{1}{2}a+\frac{1}{2}b+\frac{1}{2}c+\frac{1}{2}\right)^2=\frac{5}{16}\left(a+b+c+1\right)^2\) (Bunhiacopxki) (đpcm)

Đẳng thức xảy ra khi \(a=b=c=\frac{1}{2}\)

22 tháng 3 2019

Áp dụng bất đẳng thức Cô-si ta có:

\(\dfrac{a^2}{b^3}+\dfrac{1}{a}+\dfrac{1}{a}\ge\sqrt[3]{\dfrac{a^2}{b^3}.\dfrac{1}{a}.\dfrac{1}{a}}=\dfrac{3}{b}\)

\(\dfrac{c^2}{a^3}+\dfrac{1}{c}+\dfrac{1}{c}\ge\sqrt[3]{\dfrac{c^2}{a^3}.\dfrac{1}{c}.\dfrac{1}{c}}=\dfrac{3}{a}\)

\(\dfrac{c^2}{a^3}+\dfrac{1}{c}+\dfrac{1}{c}\ge\sqrt[3]{\dfrac{c^2}{a^3}.\dfrac{1}{c}.\dfrac{1}{c}}=\dfrac{3}{a}\)

Cộng theo vế ta được:

\(\dfrac{a^2}{b^3}+\dfrac{b^2}{c^3}+\dfrac{a^2}{a^3}+\dfrac{2}{a}+\dfrac{2}{b}+\dfrac{2}{c}\ge3\left(\dfrac{1}{a}+\dfrac{1}{b}+\dfrac{1}{c}\right)\)

\(\Leftrightarrow\dfrac{a^2}{b^3}+\dfrac{b^2}{c^3}+\dfrac{c^2}{a^3}\ge\dfrac{1}{a}+\dfrac{1}{b}+\dfrac{1}{c}\)

2 tháng 7 2019

Áp dụng Svac

\(\Sigma\frac{a^3}{b+c}=\Sigma\frac{a^4}{ab+ac}\ge\frac{\left(a^2+b^2+c^2\right)^2}{2\left(ab+bc+ca\right)}\ge\frac{\left(a^2+b^2+c^2\right)^2}{2\left(a^2+b^2+c^2\right)}=\frac{1}{2}\left(a^2+b^2+c^2\right)\)

"=" tại a=b=c

4 tháng 7 2019

E thử làm cách khác ạ:))

Không mất tính tổng quát,giả sử \(a\ge b\ge c\)

\(\Rightarrow\hept{\begin{cases}a^2\ge b^2\ge c^2\\\frac{a}{b+c}\ge\frac{b}{a+c}\ge\frac{c}{a+b}\end{cases}}\)

Áp dụng BĐT Trebysev ta có:

\(a^2\cdot\frac{a}{b+c}+b^2\cdot\frac{b}{a+c}+c^2\cdot\frac{c}{a+b}\ge\frac{a^2+b^2+c^2}{3}\cdot\left(\frac{a}{b+c}+\frac{b}{c+a}+\frac{c}{a+b}\right)\)

\(\ge\frac{a^2+b^2+c^2}{3}\cdot\frac{3}{2}\left(nesbitt\right)\)

\(=\frac{a^2+b^2+c^2}{2}\)

Dấu "=" xảy ra khi  \(a=b=c\)


 

8 tháng 2 2017

Áp dụng C-S

\(\frac{a}{2a+b+c}=\frac{a}{\left(a+b\right)+\left(a+c\right)}\le\frac{a}{4}\left(\frac{1}{a+b}+\frac{1}{a+c}\right)\)

9 tháng 2 2017

b) chính là USAMO 2004. Đây là lời giải cung cấp bởi "http://www.artofproblemsolving.com/wiki/index.php/2004_USAMO_Problems/Problem_5"

Ta chứng minh được \(x^5+1\ge x^3+x^2\) suy ra \(x^5-x^2+3\ge x^3+2\).

Ta chỉ cần CM được \(\left(a^3+1+1\right)\left(1+b^3+1\right)\left(1+1+c^3\right)\ge\left(a+b+c\right)^3\)

Nhưng đây chính là BĐT Holder cho 3 bộ số mỗi bộ 3 số.